Đến nội dung

donghaidhtt nội dung

Có 514 mục bởi donghaidhtt (Tìm giới hạn từ 29-03-2020)



Sắp theo                Sắp xếp  

#490172 Chứng minh rằng $(1+a^2)(1+b^2)(1+c^2)\geq \dfrac{125...

Đã gửi bởi donghaidhtt on 02-04-2014 - 12:29 trong Bất đẳng thức và cực trị

Đổi biến theo p. q, r. 

 

Bài này là đề thi thử ĐH nên mình muốn tìm một cách giải đơn giản, ngoài cách này ra thì bạn có cách nào khác không vậy?




#490118 Chứng minh rằng $(1+a^2)(1+b^2)(1+c^2)\geq \dfrac{125...

Đã gửi bởi donghaidhtt on 02-04-2014 - 01:21 trong Bất đẳng thức và cực trị

Cho $a,b,c$ là các số thực không âm thỏa mãn $a+b+c=\dfrac{3}{2}$

Chứng minh rằng $(1+a^2)(1+b^2)(1+c^2)\geq \dfrac{125}{64}$




#489579 $\left\{\begin{matrix} \left | xyz \right |=e...

Đã gửi bởi donghaidhtt on 30-03-2014 - 10:13 trong Phương trình - hệ phương trình - bất phương trình

Bài này chưa ai làm nhỉ?




#489111 $\left\{\begin{matrix} \left | xyz \right |=e...

Đã gửi bởi donghaidhtt on 27-03-2014 - 20:44 trong Phương trình - hệ phương trình - bất phương trình

$\left\{\begin{matrix} \left | xyz \right |=e\\ \sqrt{(\ln x^2)^2+1}+\sqrt{(\ln y^2)^2+4}+\sqrt{(\ln z^2)^2+9}=m \end{matrix}\right.$

Tìm $m$ thuộc $R$ để hệ có nghiệm $(x,y,z)$




#482523 Chứng minh MN là trung trực của OA

Đã gửi bởi donghaidhtt on 11-02-2014 - 12:11 trong Hình học

Cho (O,R). Gọi A là điểm trên (O,R). (A,r) cắt (O,R) tại B và C. Kẻ (B,r) và (C,r) cắt nhau tại A và D. Kẻ (D,DA) cắt (A,r) tại M, N. Chứng minh MN là trung trực của OA.

(Cách tìm tâm của một đường tròn cho trước chỉ bằng compa)

 




#482267 Giải các hệ phương trình sau: $\left\{\begin{m...

Đã gửi bởi donghaidhtt on 09-02-2014 - 18:55 trong Phương trình - hệ phương trình - bất phương trình

Giải các hệ phương trình sau: 

$\left\{\begin{matrix}
y+(y+1)(x+\sqrt{x^2-y^2})=44\\ 
x.(x+\sqrt{x^2-y^2})=40
\end{matrix}\right.$



#462444 HSG TỈNH TT HUẾ 2013-2014 MÔN TOÁN CHUYÊN

Đã gửi bởi donghaidhtt on 06-11-2013 - 11:39 trong Thi HSG cấp Tỉnh, Thành phố. Olympic 30-4. Đề thi và kiểm tra đội tuyển các cấp.

SỞ GIÁO DỤC VÀ ĐÀO TẠO                               KỲ THI CHỌN HỌC SINH GIỎI TỈNH

          THỪA THIÊN HUẾ                           KHỐI 12 THPT CHUYÊN _ NĂM HỌC 2013-2014

          ĐỀ CHÍNH THỨC                                               Môn: TOÁN CHUYÊN

                                                                                   Thời gian làm bài: 180 phút

                                                                                    (không kể thời gian giao đề)      

 

 

BÀI 1: (5 điểm) 

          Cho 3 điểm phân biệt $A,B,C$ thẳng hàng với $B$ nằm giữa $A$ và $C$. Đường tròn $(T)$ thay đổi và đi qua 2 điểm $B,C$. Gọi $T$ và $T^{'}$ hai tiếp điểm của hai tiếp tuyến của $(T)$ vẽ từ $A$. $M$ là trọng tâm tam giác $ATT^{'}$. Tìm $(T)$ để $MA^{2}+MB^{2}+MC^{2}$ đạt giá trị nhỏ nhất, giá trị lớn nhất.

BÀI 2: (5 điểm)

          Tìm dãy số $(U_{n})$, $n\in \mathbb{N}$ thỏa mãn đồng thời các điều kiện sau:

i/ $u_{0}=503,25$ và $u_{n}>0$

ii/ $\forall n\in \mathbb{N}, u_{n+1}\leq u_{n}$

iii/ $\forall n\in \mathbb{N^{*}}, \sum_{k=1}^{n}\frac{u_{k-1}^{2}}{u_{k}}<2013$

BÀI 3: (5 điểm)

          Cho $n$ là số nguyên dương chẵn, $a$ là số nguyên dương sao cho $a$ có đúng $n^2$ ước số lớn hơn 1. Chứng minh rằng tồn tại $m\in \mathbb{N^*},a=m^4$

BÀI 4: (5 điểm)

          Tập hợp các điểm của mặt phẳng được phân hoạch thành 3 tập hợp $A,B,C$ khác rỗng đôi một không có điểm chung.

Chứng minh rằng tồn tại một tập hợp trong 3 tập hợp $A,B,C$ chứa 3 điểm là 3 đỉnh của một tam giác sao cho tam giác đó là tam giác cân hoặc tam giác đó có số đo 3 góc tạo thành một cấp số nhân.

                                     

                                                 -------------------------Hết---------------------------

                                                    Giám thi coi thi không giải thích gì thêm




#460866 Giải pt: $\sqrt{x^2+9x-1}+x\sqrt{11-3x}=2x...

Đã gửi bởi donghaidhtt on 30-10-2013 - 16:35 trong Phương trình - hệ phương trình - bất phương trình

Biến đổi pt như sau:

$(3 x-10) (3 x-2)=0.$

Cái vế còn lại liệu có vô nghiệm ko bạn?




#454701 Giải pt: $x+e^{\dfrac{1}{x-3}}=0$

Đã gửi bởi donghaidhtt on 02-10-2013 - 19:25 trong Phương trình - hệ phương trình - bất phương trình

Giải pt: $x+e^{\dfrac{1}{x-3}}=0$




#454699 Giải pt: $x+\sqrt{x^2+1}=3^{x}$

Đã gửi bởi donghaidhtt on 02-10-2013 - 19:24 trong Phương trình - hệ phương trình - bất phương trình

Có ai giải bài này ko?




#454698 Giải pt: $\sqrt{x^2+9x-1}+x\sqrt{11-3x}=2x...

Đã gửi bởi donghaidhtt on 02-10-2013 - 19:22 trong Phương trình - hệ phương trình - bất phương trình

Ai giải chi tiết bài này đi, mình chưa tìm đc hướng làm nào cả!




#454390 Giải pt: $\sqrt{x^2+9x-1}+x\sqrt{11-3x}=2x...

Đã gửi bởi donghaidhtt on 01-10-2013 - 01:36 trong Phương trình - hệ phương trình - bất phương trình

Giải pt: $\sqrt{x^2+9x-1}+x\sqrt{11-3x}=2x+3$




#454372 Tìm GTLN của $\dfrac{1}{1-ab}+\dfrac{...

Đã gửi bởi donghaidhtt on 30-09-2013 - 22:18 trong Bất đẳng thức và cực trị

Cho $a,b,c$ là 3 số thực không âm thỏa mãn $a^2+b^2+c^2=1$

Tìm GTLN của $\dfrac{1}{1-ab}+\dfrac{1}{1-bc}+\dfrac{1}{1-ca}$




#454370 Giải pt: $x+\sqrt{x^2+1}=3^{x}$

Đã gửi bởi donghaidhtt on 30-09-2013 - 22:15 trong Phương trình - hệ phương trình - bất phương trình

Giải pt: $x+\sqrt{x^2+1}=3^{x}$




#454368 Cho x,y thỏa mãn $8x^2+y^2+\dfrac{1}{4x^2}=4...

Đã gửi bởi donghaidhtt on 30-09-2013 - 22:14 trong Bất đẳng thức và cực trị

Cho x,y thỏa mãn $8x^2+y^2+\dfrac{1}{4x^2}=4$

Tìm GTNN của $xy$




#453058 Giải các phương trình:1) $16x+30\sqrt{1-x^2}=17(\sqr...

Đã gửi bởi donghaidhtt on 25-09-2013 - 23:34 trong Phương trình - hệ phương trình - bất phương trình

Giải các phương trình:

1) $16x+30\sqrt{1-x^2}=17(\sqrt{1-x}+\sqrt{1+x})$

2) $125y^5-125y^3+6\sqrt{15}=0, y\in \begin{bmatrix} -1;1 \end{bmatrix}$




#451779 Tìm quỹ tích trọng tâm $G$ của tam giác $AHK$

Đã gửi bởi donghaidhtt on 20-09-2013 - 09:08 trong Hình học

Cho đường tròn tâm $O$ đường kính $AB=2R$, 1 cung tròn $MN$ có độ dài $R$ di động trên đường tròn. $AM$ cắt $BN$ tại $H$, $AN$ cắt $BM$ tại $K$. Tìm quỹ tích trọng tâm $G$ của tam giác $AHK$




#445797 Cho x,y,z là các số thực dương với $3x^2+4y^2+5z^2=2xyz$ Tìm Min...

Đã gửi bởi donghaidhtt on 27-08-2013 - 22:13 trong Bất đẳng thức và cực trị

Cho x,y,z là các số thực dương với $3x^2+4y^2+5z^2=2xyz$

Tìm Min $P=3x+2y+z$




#445456 Tìm GTLN,GTNN của biểu thức $P =xy^2(8-x-y)$

Đã gửi bởi donghaidhtt on 25-08-2013 - 21:57 trong Đại số

min

$P= 4x\frac{y}{2}\frac{y}{2}\left ( 8-x-y \right )$

ta có $x+y\leq 12\Rightarrow 8-x-y\geq -4$

$x\frac{y}{2}\frac{y}{2}\leq \left ( x+\frac{y}{2}+\frac{y}{2} \right )^{3}/27\leq \frac{12^{3}}{27}$

$\Rightarrow P\geq 4\frac{12^{3}}{27}.(-4)= -4^{5}$

Cái này hình như là sai phải không? Bạn xem lại thử?




#444476 Tính thể tích khối tứ diện $ABCD$ và khoảng cách giữa 2 đường thẳng...

Đã gửi bởi donghaidhtt on 21-08-2013 - 11:46 trong Hình học không gian

 

Giải
Gọi M, N lần lượt là trung điểm của BC và BD.
- Do $\triangle$ ABD cân tại A $\Rightarrow$ AN $\perp$ BD
- M, N là trung điểm BC, BD $\Rightarrow $ MN // DC $\Rightarrow$ MN $\perp$ BD
Vậy: BD $\perp$ (AMN) $\Rightarrow$ BD $\perp$ AM
 
Mặt khác: $\triangle$ ABC cân tại A $\Rightarrow$ AM $\perp$ BC
Suy ra: AM $\perp$ (BCD) hay AM là đường cao của tứ diện.

Mình có ý kiến về đoạn này, có thể chứng minh ngắn hơn: $\left\{\begin{matrix} AD=AB=AC\\ MB=MC=MD \end{matrix}\right.$

nên $AM$ là trục đường tròn ngoại tiếp tam giác $BCD$. Từ đó $AM$ vuông với $(BCD)$




#444151 (SBC) vuông với (ADH)

Đã gửi bởi donghaidhtt on 19-08-2013 - 21:50 trong Hình học không gian

BC vuông $AM$

Mà $AM$ thuộc $mp(ADH)$ $\Rightarrow$ $mp(SBC)$ vuông góc $mp(ADH)$ (đpcm)

 

Mình nghĩ là cái này bị sai? Bạn giải thích rõ hơn được không?




#443997 $f'(x) > 0, \forall x \in (a;b)$ thì hàm số...

Đã gửi bởi donghaidhtt on 19-08-2013 - 12:02 trong Hàm số - Đạo hàm

Sách ghi:
" Nếu hàm số f liên tục trên đoạn [a;b] vào có đạo hàm f'(x) > 0 trên khoảng (a;b) thì hàm số f đồng biến trên đoạn [a;b] "
Sao xác định đạo hàm trên khoảng nhưng lại kết luận hàm số ở trên đoạn vậy? Trường hợp x= a , x=b f'(x)<0 thì sao?
Tường tự như vậy cũng ngay trang đó VD1:
Chứng minh rằng hàm số f(x) = √(1- x^2) nghịch biến trên đoạn [0;1].
Sách giải: f'(x) < 0 với mọi x ϵ (0;1) . Do đó hàm số nb trên đoạn [0;1].
Mong các anh chị giải đáp giúp! cám ơn nhiều!

Bạn xem cách chứng minh ở đây nhé!

File gửi kèm




#443712 $\left\{\begin{matrix} e^{x}+...

Đã gửi bởi donghaidhtt on 17-08-2013 - 20:00 trong Phương trình - hệ phương trình - bất phương trình

$2012=e^{x}+\frac{x}{\sqrt{x^{2}-1}}$(1)

vậy có nghĩa hàm f(x) chỉ có 1 cực tiểu suy ra (1) có 2 nghiệm phân biệt.

Hàm số chỉ có 1 cực tiểu thì chắc gì pt (1) đã có 2 nghiệm phân biệt hả bạn? Có thể vô nghiệm hoặc có 1 nghiệm nếu $2012 \leq$ điểm cực tiểu (hay $\leq f(x)$ với mọi $x>1$)




#436730 Tìm max: $\sqrt{x-x^3}$+$\sqrt{x+x^3...

Đã gửi bởi donghaidhtt on 21-07-2013 - 00:46 trong Bất đẳng thức và cực trị



Tìm max: $\sqrt{x-x^3}$+$\sqrt{x+x^3}$ biết 0$\leq$x$\leq$1

THPT:

Đặt $f(x)=\sqrt{x-x^3}$+$\sqrt{x+x^3}$

Đạo hàm $f'(x)=\dfrac{1-3x^2}{2\sqrt{x-x^3}}+\dfrac{1+3x^2}{2\sqrt{x+x^3}},\forall x\in (0;1)$

$f'(x)=0$

$\Leftrightarrow \dfrac{1-3x^2}{2\sqrt{x-x^3}}+\dfrac{1+3x^2}{2\sqrt{x+x^3}}=0$

$\Leftrightarrow (1-3x^2)\sqrt{x+x^3}+(1+3x^2)\sqrt{x-x^3}=0$

$\Leftrightarrow \sqrt{x+x^3}+\sqrt{x-x^3}=3x^2(\sqrt{x+x^3}-\sqrt{x-x^3})$

$\Leftrightarrow 3x^2(x+x^3-x+x^3)=(\sqrt{x+x^3}+\sqrt{x-x^3})^2$

$\Leftrightarrow 6x^5=2x+2x\sqrt{1-x^4}$

$\Leftrightarrow 3x^4-1=\sqrt{1-x^4}$

$\Leftrightarrow x=\sqrt[4]{\frac{5}{9}}$

Dựa vào bảng biến thiên ta có $f_{max}=f_{(\sqrt[4]{\dfrac{5}{9}})}$




#436726 Tìm tâm I của đường tròn nội tiếp tam giác ABC

Đã gửi bởi donghaidhtt on 21-07-2013 - 00:31 trong Phương pháp tọa độ trong mặt phẳng

Làm ơn cm KC = KI !

$KI=KB\Leftrightarrow \widehat{KBI}=\widehat{KIB}\Leftrightarrow \widehat{KBC}+\widehat{CBI}=\widehat{ABI}+\widehat{BAI}\Leftrightarrow \widehat{KBC}=\widehat{BAI}\Leftrightarrow \widehat{KBC}=\widehat{KCB}$

Đúng do tam giác $KBC$ vuông cân tại $K$